Exponential function of a Hermitian matrix

The name of the pictureThe name of the pictureThe name of the pictureClash Royale CLAN TAG#URR8PPP











up vote
2
down vote

favorite












Given



$$H = beginpmatrixsin theta & 0 & cos theta \ 0 & 1 & 0 \ cos theta & 0 & -sin theta endpmatrix$$



where $theta=pi/6$, then what is $exp left( i fracpi2 H right)$?



I tried to calculate in the following way
$e^(ipi H)/2=[e^(ipi/2)]^H=i^H$. I do not know how to proceed.







share|cite|improve this question

















  • 1




    You need to diagonalize $H,$ if possible. Then you can use the Taylor series expansion of the exponential to finish.
    – Adrian Keister
    Jul 25 at 15:51










  • @AdrianKeister As it's a reflection matrix, it's quite easy to diagonalise, even over $Bbb R$.
    – Lord Shark the Unknown
    Jul 25 at 15:57










  • @LordSharktheUnknown: Right most indubitably ho.
    – Adrian Keister
    Jul 25 at 16:04














up vote
2
down vote

favorite












Given



$$H = beginpmatrixsin theta & 0 & cos theta \ 0 & 1 & 0 \ cos theta & 0 & -sin theta endpmatrix$$



where $theta=pi/6$, then what is $exp left( i fracpi2 H right)$?



I tried to calculate in the following way
$e^(ipi H)/2=[e^(ipi/2)]^H=i^H$. I do not know how to proceed.







share|cite|improve this question

















  • 1




    You need to diagonalize $H,$ if possible. Then you can use the Taylor series expansion of the exponential to finish.
    – Adrian Keister
    Jul 25 at 15:51










  • @AdrianKeister As it's a reflection matrix, it's quite easy to diagonalise, even over $Bbb R$.
    – Lord Shark the Unknown
    Jul 25 at 15:57










  • @LordSharktheUnknown: Right most indubitably ho.
    – Adrian Keister
    Jul 25 at 16:04












up vote
2
down vote

favorite









up vote
2
down vote

favorite











Given



$$H = beginpmatrixsin theta & 0 & cos theta \ 0 & 1 & 0 \ cos theta & 0 & -sin theta endpmatrix$$



where $theta=pi/6$, then what is $exp left( i fracpi2 H right)$?



I tried to calculate in the following way
$e^(ipi H)/2=[e^(ipi/2)]^H=i^H$. I do not know how to proceed.







share|cite|improve this question













Given



$$H = beginpmatrixsin theta & 0 & cos theta \ 0 & 1 & 0 \ cos theta & 0 & -sin theta endpmatrix$$



where $theta=pi/6$, then what is $exp left( i fracpi2 H right)$?



I tried to calculate in the following way
$e^(ipi H)/2=[e^(ipi/2)]^H=i^H$. I do not know how to proceed.









share|cite|improve this question












share|cite|improve this question




share|cite|improve this question








edited Jul 25 at 15:55









Rodrigo de Azevedo

12.6k41751




12.6k41751









asked Jul 25 at 15:49









Purushothaman

1866




1866







  • 1




    You need to diagonalize $H,$ if possible. Then you can use the Taylor series expansion of the exponential to finish.
    – Adrian Keister
    Jul 25 at 15:51










  • @AdrianKeister As it's a reflection matrix, it's quite easy to diagonalise, even over $Bbb R$.
    – Lord Shark the Unknown
    Jul 25 at 15:57










  • @LordSharktheUnknown: Right most indubitably ho.
    – Adrian Keister
    Jul 25 at 16:04












  • 1




    You need to diagonalize $H,$ if possible. Then you can use the Taylor series expansion of the exponential to finish.
    – Adrian Keister
    Jul 25 at 15:51










  • @AdrianKeister As it's a reflection matrix, it's quite easy to diagonalise, even over $Bbb R$.
    – Lord Shark the Unknown
    Jul 25 at 15:57










  • @LordSharktheUnknown: Right most indubitably ho.
    – Adrian Keister
    Jul 25 at 16:04







1




1




You need to diagonalize $H,$ if possible. Then you can use the Taylor series expansion of the exponential to finish.
– Adrian Keister
Jul 25 at 15:51




You need to diagonalize $H,$ if possible. Then you can use the Taylor series expansion of the exponential to finish.
– Adrian Keister
Jul 25 at 15:51












@AdrianKeister As it's a reflection matrix, it's quite easy to diagonalise, even over $Bbb R$.
– Lord Shark the Unknown
Jul 25 at 15:57




@AdrianKeister As it's a reflection matrix, it's quite easy to diagonalise, even over $Bbb R$.
– Lord Shark the Unknown
Jul 25 at 15:57












@LordSharktheUnknown: Right most indubitably ho.
– Adrian Keister
Jul 25 at 16:04




@LordSharktheUnknown: Right most indubitably ho.
– Adrian Keister
Jul 25 at 16:04










3 Answers
3






active

oldest

votes

















up vote
3
down vote



accepted










The eigenvalues of this matrix are $1$ (double) and $-1$ (simple).
As it is real symmetric, it is diagonalisable. If $v$ is an eigenvector
then $Hv=pm v$. Therefore
$$exp(pi i H/2)v=exp(pmpi i/2)v
=pm iv=iHv.$$
Hence $exp(pi iH/2)=iH$.






share|cite|improve this answer






























    up vote
    2
    down vote













    Note that $H^2=I$, thus $$ e^tH = I + tH +t^2/2!I +t^3/3!H +... $$



    $$= (1+ t^2/2! +t^4/4!+...)I + (t +t^3/3! + t^5/5!+...)H $$



    $$= (sinh t)I + (cosh t)H$$



    For $t=(pi/2) i$ we get $$ sinh t=0 $$ and $$ cosh t=i$$ thus $$e^(pi/2) iH = iH$$






    share|cite|improve this answer




























      up vote
      1
      down vote













      The eigenvalues of $H$ are $lambda =1,1,-1$



      According to Cayley-Hamilton Theorem $$ e^tH=alpha I +beta H + gamma H^2$$ where $ alpha,beta,gamma$ are functions of t to be found by the equation $$ e^tlambda=alpha +beta lambda + gamma lambda^2 $$ and its derivative with repect to $lambda.$



      We find $$ beta = (e^t+e^-t)/2, gamma = 1/2te^t - beta /2, alpha = e^t-beta - gamma $$



      For $t=ipi /2$ we have $$ beta = i, gamma = -pi /4, alpha = pi/4 $$



      Thus we get $$e^(ipi /2) H=iH$$






      share|cite|improve this answer





















        Your Answer




        StackExchange.ifUsing("editor", function ()
        return StackExchange.using("mathjaxEditing", function ()
        StackExchange.MarkdownEditor.creationCallbacks.add(function (editor, postfix)
        StackExchange.mathjaxEditing.prepareWmdForMathJax(editor, postfix, [["$", "$"], ["\\(","\\)"]]);
        );
        );
        , "mathjax-editing");

        StackExchange.ready(function()
        var channelOptions =
        tags: "".split(" "),
        id: "69"
        ;
        initTagRenderer("".split(" "), "".split(" "), channelOptions);

        StackExchange.using("externalEditor", function()
        // Have to fire editor after snippets, if snippets enabled
        if (StackExchange.settings.snippets.snippetsEnabled)
        StackExchange.using("snippets", function()
        createEditor();
        );

        else
        createEditor();

        );

        function createEditor()
        StackExchange.prepareEditor(
        heartbeatType: 'answer',
        convertImagesToLinks: true,
        noModals: false,
        showLowRepImageUploadWarning: true,
        reputationToPostImages: 10,
        bindNavPrevention: true,
        postfix: "",
        noCode: true, onDemand: true,
        discardSelector: ".discard-answer"
        ,immediatelyShowMarkdownHelp:true
        );



        );








         

        draft saved


        draft discarded


















        StackExchange.ready(
        function ()
        StackExchange.openid.initPostLogin('.new-post-login', 'https%3a%2f%2fmath.stackexchange.com%2fquestions%2f2862551%2fexponential-function-of-a-hermitian-matrix%23new-answer', 'question_page');

        );

        Post as a guest






























        3 Answers
        3






        active

        oldest

        votes








        3 Answers
        3






        active

        oldest

        votes









        active

        oldest

        votes






        active

        oldest

        votes








        up vote
        3
        down vote



        accepted










        The eigenvalues of this matrix are $1$ (double) and $-1$ (simple).
        As it is real symmetric, it is diagonalisable. If $v$ is an eigenvector
        then $Hv=pm v$. Therefore
        $$exp(pi i H/2)v=exp(pmpi i/2)v
        =pm iv=iHv.$$
        Hence $exp(pi iH/2)=iH$.






        share|cite|improve this answer



























          up vote
          3
          down vote



          accepted










          The eigenvalues of this matrix are $1$ (double) and $-1$ (simple).
          As it is real symmetric, it is diagonalisable. If $v$ is an eigenvector
          then $Hv=pm v$. Therefore
          $$exp(pi i H/2)v=exp(pmpi i/2)v
          =pm iv=iHv.$$
          Hence $exp(pi iH/2)=iH$.






          share|cite|improve this answer

























            up vote
            3
            down vote



            accepted







            up vote
            3
            down vote



            accepted






            The eigenvalues of this matrix are $1$ (double) and $-1$ (simple).
            As it is real symmetric, it is diagonalisable. If $v$ is an eigenvector
            then $Hv=pm v$. Therefore
            $$exp(pi i H/2)v=exp(pmpi i/2)v
            =pm iv=iHv.$$
            Hence $exp(pi iH/2)=iH$.






            share|cite|improve this answer















            The eigenvalues of this matrix are $1$ (double) and $-1$ (simple).
            As it is real symmetric, it is diagonalisable. If $v$ is an eigenvector
            then $Hv=pm v$. Therefore
            $$exp(pi i H/2)v=exp(pmpi i/2)v
            =pm iv=iHv.$$
            Hence $exp(pi iH/2)=iH$.







            share|cite|improve this answer















            share|cite|improve this answer



            share|cite|improve this answer








            edited Jul 25 at 16:07









            Rodrigo de Azevedo

            12.6k41751




            12.6k41751











            answered Jul 25 at 16:02









            Lord Shark the Unknown

            84.9k950111




            84.9k950111




















                up vote
                2
                down vote













                Note that $H^2=I$, thus $$ e^tH = I + tH +t^2/2!I +t^3/3!H +... $$



                $$= (1+ t^2/2! +t^4/4!+...)I + (t +t^3/3! + t^5/5!+...)H $$



                $$= (sinh t)I + (cosh t)H$$



                For $t=(pi/2) i$ we get $$ sinh t=0 $$ and $$ cosh t=i$$ thus $$e^(pi/2) iH = iH$$






                share|cite|improve this answer

























                  up vote
                  2
                  down vote













                  Note that $H^2=I$, thus $$ e^tH = I + tH +t^2/2!I +t^3/3!H +... $$



                  $$= (1+ t^2/2! +t^4/4!+...)I + (t +t^3/3! + t^5/5!+...)H $$



                  $$= (sinh t)I + (cosh t)H$$



                  For $t=(pi/2) i$ we get $$ sinh t=0 $$ and $$ cosh t=i$$ thus $$e^(pi/2) iH = iH$$






                  share|cite|improve this answer























                    up vote
                    2
                    down vote










                    up vote
                    2
                    down vote









                    Note that $H^2=I$, thus $$ e^tH = I + tH +t^2/2!I +t^3/3!H +... $$



                    $$= (1+ t^2/2! +t^4/4!+...)I + (t +t^3/3! + t^5/5!+...)H $$



                    $$= (sinh t)I + (cosh t)H$$



                    For $t=(pi/2) i$ we get $$ sinh t=0 $$ and $$ cosh t=i$$ thus $$e^(pi/2) iH = iH$$






                    share|cite|improve this answer













                    Note that $H^2=I$, thus $$ e^tH = I + tH +t^2/2!I +t^3/3!H +... $$



                    $$= (1+ t^2/2! +t^4/4!+...)I + (t +t^3/3! + t^5/5!+...)H $$



                    $$= (sinh t)I + (cosh t)H$$



                    For $t=(pi/2) i$ we get $$ sinh t=0 $$ and $$ cosh t=i$$ thus $$e^(pi/2) iH = iH$$







                    share|cite|improve this answer













                    share|cite|improve this answer



                    share|cite|improve this answer











                    answered Jul 25 at 17:37









                    Mohammad Riazi-Kermani

                    27.4k41852




                    27.4k41852




















                        up vote
                        1
                        down vote













                        The eigenvalues of $H$ are $lambda =1,1,-1$



                        According to Cayley-Hamilton Theorem $$ e^tH=alpha I +beta H + gamma H^2$$ where $ alpha,beta,gamma$ are functions of t to be found by the equation $$ e^tlambda=alpha +beta lambda + gamma lambda^2 $$ and its derivative with repect to $lambda.$



                        We find $$ beta = (e^t+e^-t)/2, gamma = 1/2te^t - beta /2, alpha = e^t-beta - gamma $$



                        For $t=ipi /2$ we have $$ beta = i, gamma = -pi /4, alpha = pi/4 $$



                        Thus we get $$e^(ipi /2) H=iH$$






                        share|cite|improve this answer

























                          up vote
                          1
                          down vote













                          The eigenvalues of $H$ are $lambda =1,1,-1$



                          According to Cayley-Hamilton Theorem $$ e^tH=alpha I +beta H + gamma H^2$$ where $ alpha,beta,gamma$ are functions of t to be found by the equation $$ e^tlambda=alpha +beta lambda + gamma lambda^2 $$ and its derivative with repect to $lambda.$



                          We find $$ beta = (e^t+e^-t)/2, gamma = 1/2te^t - beta /2, alpha = e^t-beta - gamma $$



                          For $t=ipi /2$ we have $$ beta = i, gamma = -pi /4, alpha = pi/4 $$



                          Thus we get $$e^(ipi /2) H=iH$$






                          share|cite|improve this answer























                            up vote
                            1
                            down vote










                            up vote
                            1
                            down vote









                            The eigenvalues of $H$ are $lambda =1,1,-1$



                            According to Cayley-Hamilton Theorem $$ e^tH=alpha I +beta H + gamma H^2$$ where $ alpha,beta,gamma$ are functions of t to be found by the equation $$ e^tlambda=alpha +beta lambda + gamma lambda^2 $$ and its derivative with repect to $lambda.$



                            We find $$ beta = (e^t+e^-t)/2, gamma = 1/2te^t - beta /2, alpha = e^t-beta - gamma $$



                            For $t=ipi /2$ we have $$ beta = i, gamma = -pi /4, alpha = pi/4 $$



                            Thus we get $$e^(ipi /2) H=iH$$






                            share|cite|improve this answer













                            The eigenvalues of $H$ are $lambda =1,1,-1$



                            According to Cayley-Hamilton Theorem $$ e^tH=alpha I +beta H + gamma H^2$$ where $ alpha,beta,gamma$ are functions of t to be found by the equation $$ e^tlambda=alpha +beta lambda + gamma lambda^2 $$ and its derivative with repect to $lambda.$



                            We find $$ beta = (e^t+e^-t)/2, gamma = 1/2te^t - beta /2, alpha = e^t-beta - gamma $$



                            For $t=ipi /2$ we have $$ beta = i, gamma = -pi /4, alpha = pi/4 $$



                            Thus we get $$e^(ipi /2) H=iH$$







                            share|cite|improve this answer













                            share|cite|improve this answer



                            share|cite|improve this answer











                            answered Jul 25 at 17:01









                            Mohammad Riazi-Kermani

                            27.4k41852




                            27.4k41852






















                                 

                                draft saved


                                draft discarded


























                                 


                                draft saved


                                draft discarded














                                StackExchange.ready(
                                function ()
                                StackExchange.openid.initPostLogin('.new-post-login', 'https%3a%2f%2fmath.stackexchange.com%2fquestions%2f2862551%2fexponential-function-of-a-hermitian-matrix%23new-answer', 'question_page');

                                );

                                Post as a guest













































































                                Comments

                                Popular posts from this blog

                                Color the edges and diagonals of a regular polygon

                                Relationship between determinant of matrix and determinant of adjoint?

                                What is the equation of a 3D cone with generalised tilt?